3x+5=8(x-2)+1
Solve the following equation for x

Answers

Answer 1

Answer: x=4

Step-by-step explanation:

1. 3x+5 = 8x-16+1

2. 3x+5 = 8x-15

3. 3x+20 = 8x

4. 20 = 5x

5. x = 4


Related Questions

A fisherman drops a fishing line into the sea. The end of the fishing pole is at an elevation of 5 feet. The hook that is in the water is at an elevation of -2 feet.cessmentThe number line shows their heights. Sea level is represented by 0.1. Write an absolute value expression telling how many feet the end of the fishingpole is above sea level. Evaluate the expression.2. Write an absolute value expression telling how many feet the hook is below sealevel. Evaluate the expression. 3. If the fishing line goes straight down into the water, what is the distance betweenthe end of the pole and the hook? Explain how you found this distance.

Answers

In this case, we'll have to carry out several steps to find the solution.

Step 01:

Data:

sea level = 0 ft

end of fishing pole = 5 ft

hook = -2 ft

Step 02:

absolute value:

distance between sea level and the end of fishing pole:

| 5 - 0| = | 5 | = 5 ft

distance between hook and sea level:

|0 - (-2)| = | 0 + 2| = |2| = 2 ft

distance between hook and the end of the fishing pole:

| 5 - (-2)| = | 5 + 2| = |7| = 7 ft

To find out the distance we must consider the entire interval.

That is the full solution.

Answer:

Given a fishing line acting as number line, find the asked distances

Explanation:

given a fishing line having its one end of the fishing pole above the water. Let this distance be denoted by 'a'.

given that the hook of this fishing line is in the water hence, below the sea level. Let this depth be denoted by 'b'.

let the height of pole from sea-level be denoted by  , height of the hook from sea level be denoted by  and the length between pole end and hook be

since, this fishing line is acting as a number line with sea level as . The depth of fishing hook is negative  and the elevation of the pole end is positive .  

hence we get expressions,                                                                                            

for given values   the evaluation of the expressions is,              

Step-by-step explanation:

the circle below is centered at the point (2,-1 ) and had a radius of length 3 what is its equation

Answers

The standard equation for a circle is

[tex]\begin{gathered} (x-a)^2+(y-b)^2=r^2 \\ \text{where} \\ a=2 \\ b=-1 \\ r=\text{radius}=3 \\ (x-2)^2+(y-(-1))^2=3^2 \\ (x-2)^2+(y+1)^2=3^2 \\ \end{gathered}[/tex]

jen has to put 180 cards into boxes of 6 cards each. she put 150 cards into boxes. write an equation that could use to figure out how many boxes jen need. let b stand for the unknown number of boxes.

Answers

Let b be the number of boxes.

Since each box has 6 cards, we will have the term 6b to get the remaining boxes.

Since Jen already put 150 cards into boxes, we have the following:

[tex]150+6b=180[/tex]

for 150 cards, Jen used 25 boxes. We can check that the remaining 5 boxes can be found using the previous equation:

[tex]\begin{gathered} 150+6b=180 \\ \Rightarrow6b=180-150=30 \\ \Rightarrow b=\frac{30}{6}=5 \\ b=5 \end{gathered}[/tex]

therefore, the equation is 150+6b=180

Which answer shows how to solve the given equation using the quadratic formula? 22 - 3. - 4= 0 3+, 22-4(2)(-4) 2(2) -(-3)=1/(-3)2-4(2)(-4) 2(2) 4+/(-3) -4(2)(-4) 2 3+1/32-4(-3)(-4) 2(2)

Answers

hello

the question here is a given quadratic equation and we're required to use quadratic formula to solve it.

[tex]2x^2-3x-4=0[/tex]

now, to solve this, let's bring out quadratic formula first

[tex]x=-b\pm\frac{\sqrt[]{b^2-4ac}}{2a}[/tex]

now from our equation given, we can easily identify a, b and c.

[tex]\begin{gathered} 2x^2-3x-4=0 \\ a=2 \\ b=-3 \\ c=-4 \end{gathered}[/tex]

next we plug in the variables into the equation and solve

[tex]undefined[/tex]

Miss Taylor drove 30 miles in March she drove 9 times as many miles in May as she did in March she drove 2 times as many miles in April as she did in May how many miles did Miss Taylor Drive in April.

Answers

then we use the statement to solve

Miss Taylor drove 30 miles in March

[tex]March=30[/tex]

she drove 9 times as many miles in May as she did in March

[tex]\begin{gathered} May=9\text{March} \\ May=9\times30=270 \end{gathered}[/tex]

she drove 2 times as many miles in April as she did in May

[tex]\begin{gathered} April=2May \\ April=2\times270=540 \end{gathered}[/tex]

Taylor Drove 540 Miles in April

According to the Florida Agency for Workforce, the monthly average number of unemployment claims in a certain county is given by () = 22.16^2 − 238.5 + 2005, where t is the number of years after 1990. a) During what years did the number of claims decrease? b) Find the relative extrema and interpret it.

Answers

SOLUTION

(a) Now from the question, we want to find during what years the number of claims decrease. Let us make the graph of the function to help us answer this

[tex]N(t)=22.16^2-238.5t+2005[/tex]

We have

From the graph above, we can see that the function decreased at between x = 0 to x = 5.381

Hence the number of claims decreased between 1990 to 1995, that is 1990, 1991, 1992, 1993, 1994 and 1995

Note that 1990 was taken as zero

(b) The relative extrema from the graph is at 5.381, which represents 1995.

Hence the interpretation is that it is at 1995 that the minimum number of claims is approximately 1363.

Note that 1363 is approximately the y-value 1363.278

The director of a film festival received 9 submissions, 7 of which were sci-fi films. If the director randomly chose to play 6 of the submissions on the first day of the festival, what is the probability that all of them are sci-fi films? Write your answer as a decimal rounded to four decimal places .

Answers

Given data:

9 submissions out of which 7 were sci-fi

If the director randomly chose to play 6 of the submissions on the first day of the festival

Then, the probability that all of them are sci-fi films will be obtained as follows

At the first selection, it will be: 7/9

At the second selection, it will be: 6/8

At the third selection, it will be: 5/7

At the fourth selection, it will be: 4/6

At the fifth selection, it will be: 3/5

At the sixth selection, it will be: 2/4

Thus, the probability will be

[tex]\frac{7}{9}\times\frac{6}{8}\times\frac{5}{7}\times\frac{4}{6}\times\frac{3}{5}\times\frac{2}{4}=\frac{5040}{60480}[/tex]

=>

[tex]\frac{5040}{60480}=\frac{1}{12}[/tex]

=>

[tex]\frac{1}{12}=0.0833[/tex]

Answer = 0.0833

65+ (blank) =180

11x + (blank)=180

11x =

x =

Answers

Answer:

sorry if this is wrong

I just answered it according to the question you gave not the pic

Step-by-step explanation:

x = 65

11x + x = 180

12x = 180

x = 180 ÷ 12

= 15

2x - 6(x-3) ≥ 5

solve for x.

Answers

Answer:

It’s siu

Step-by-step explanation:

Answer:x≤4.6

Step-by-step explanation: 2x-6(x-3)≥5. 1).combine the like terms. 2x+x=3x & -6+-3=-9. 2). isolate the "x". 3x-9≥5. 3x≥14. 3). divide both sides by your coefficient. 3x≥14/ 3

x≥4.6

4) flip your sign. x≤4.6

According to projections through the year 2030, the population y of the given state in year x is approximated byState A: - 5x + y = 11,700State B: - 144x + y = 9,000where x = 0 corresponds to the year 2000 and y is in thousands. In what year do the two states have the same population?The two states will have the same population in the year

Answers

The x variable represents the year in question. The year 2000 is represented by x = 0, 2001 would be repreented by x = 1, and so on.

The year in which both states would have the same population can be determined by the value of x which satisfies both equations.

We would now solve these system of equations as follows;

[tex]\begin{gathered} -5x+y=11700---(1) \\ -144x+y=9000---(2) \\ \text{Subtract equation (2) from equation (1);} \\ -5x-\lbrack-144x\rbrack=11700-9000 \\ -5x+144x=2700 \\ 139x=2700 \\ \text{Divide both sides by 139} \\ x=19.4244 \\ x\approx19\text{ (rounded to the nearest whole number)} \end{gathered}[/tex]

Note that x = 19 represents the year 2019

ANSWER:

The two states will have the same population in the year 2019

solving systems by graphing and tables : equations and inequalities

Answers

Given,

The system of inequalitites are,

[tex]\begin{gathered} 2x+3y>0 \\ x-y\leq5 \end{gathered}[/tex]

The graph of the inequalities is,

The are three possible solution for the inequality.

For (0, 0),

[tex]\begin{gathered} 2x+3y>0 \\ 2(0)+3(0)>0 \\ 0>0 \\ \text{Similarly,} \\ x-y\leq5 \\ 0-0\leq5 \\ 0\leq5 \end{gathered}[/tex]

For (3, -2),

[tex]\begin{gathered} 2x+3y>0 \\ 2(3)+3(-2)>0 \\ 0>0 \\ \text{Similarly,} \\ x-y\leq5 \\ 3-(-2)\leq5 \\ 5=5 \end{gathered}[/tex]

For (5, 0),

[tex]\begin{gathered} 2x+3y>0 \\ 2(5)+3(0)>0 \\ 5>0 \\ \text{Similarly,} \\ x-y\leq5 \\ 5-(0)\leq5 \\ 5=5 \end{gathered}[/tex]

Hence, the solution of the inequalities is (5, 0).

Find the lateral surface area of thiscylinder. Round to the nearest tenth.8ft4ftLSA = [ ? ] ft2—

Answers

Solution

Step 1:

Write the lateral surface area or curved surface area of a cylinder:

[tex]Lateral\text{ surface area = 2}\pi rh[/tex]

Step 2:

Write the given data

Height h = 8ft

Radius r = 4 ft

Step 3:

Substitute in the formula to find the lateral surface area.

[tex]\begin{gathered} Lateral\text{ surface area = 2}\pi rh \\ =\text{ 2 }\times\text{ 3.142 }\times\text{ 4 }\times\text{ 8} \\ =\text{ 201.1 ft}^2 \end{gathered}[/tex]

Final answer

201.1

MP is the perpendicular bisector of the side AC of the triangle ABC, in which AB = AC. prove that angle APB = 2 angle B

Answers

We have the following:

[tex]\begin{gathered} \frac{a}{\sin now,[tex]\begin{gathered}

Let the graph of f(x) be given below. Find the formula of f(x), a polynomial function, of least degree.

Answers

[tex]f(x)\text{ = }\frac{1}{2}x(x\text{ }+2)(x-4)[/tex]

Explanation:

To detrmine the formula of the polynomial, we check for the roots on the graph:

when y = 0, x = -2

when y = 0, x = 4

We have two roots.

x = -2

x + 2 = 0

x = 4

x - 4 = 0

3rd factor is x = 0

Hence, we have two factors: x(x + 2) and (x - 4)

The polynomial function using the factors:

[tex]f(x)\text{ = ax(x + 2)(x - 4)}[/tex]

Next, we find the value of a:

To get a , we pick a point on the graph. let the point be (0, -4)

substitute the point in the function above:

[tex]\begin{gathered} f(x)\text{ = y = -4, x = 0} \\ -4\text{ = a(0 + 2) (0 - 4)} \\ -4\text{ = a(2)(-4)} \\ -4\text{ = -8a} \\ a\text{ = -4/-8} \\ a\text{ = 1/2} \end{gathered}[/tex]

The formula of the polynomial becomes:

[tex]f(x)\text{ = }\frac{1}{2}x(x\text{ }+2)(x-4)[/tex]

In a scale drawing of a rectangularswimming pool, the scale is 2 inch: 4feet. Find the perimeter and area ofthe swimming pool.15 in.3.5 in.

Answers

The given scale is

[tex]2in\colon4ft[/tex]

This means each two inches of the scale represents 4 feet of the actual size (or each inch is equivalent to two feet).

So, if the dimensions of the scale are 15 inches by 3.5 inches, then the actual dimensions would be 30 feet by 7 feet.

The perimeter would be

[tex]P=2(w+l)=2(30+7)=2(37)=74ft[/tex]

The area would be

[tex]A=w\cdot l=30.7=210ft^2[/tex]

Therefore, the perimeter is 74 feet, and the area is 210 square feet.

I don’t know how to find the value of x. Geometry is so confusing too me, i can never understand it no matter how many times i re-read my instructions.

Answers

The value of x = 40°

Explanation:

To solve for x, we will use an illustration:

When two lines intersect, the angles opposite each other are vertical angles. Vertical angles are equal.

The angles marked in magenta are equal.

The angle by the right in magenta colour will also be 52°.

The sum of angles in a triangle = 180°

x° + 52° + 88° = 180°

x + 140 = 180

subtract 140 from both sides:

x + 140 - 140 = 180 - 140

x = 40°

this is a Statistics question. Please help

Answers

Using the normal distribution, it is found that the measures are given as follows:

a) Proportion with less than 125 mg/dl: 0.16.

b) Percentage between 200 and 225 mg/dl: 2.35%.

Normal Probability Distribution

The z-score of a measure X of a variable that has mean [tex]\mu[/tex] and standard deviation [tex]\sigma[/tex] is given by the rule presented as follows:

[tex]Z = \frac{X - \mu}{\sigma}[/tex]

The z-score measures how many standard deviations the measure X is above or below the mean of the distribution, depending if the z-score is positive or negative.From the z-score table, the p-value associated with the z-score is found, and it represents the percentile of the measure X in the distribution.

The mean and the standard deviation of the cholesterol levels are given as follows:

[tex]\mu = 150, \sigma = 25[/tex]

The proportion below 125 mg/dl is the p-value of Z when X = 125, hence:

[tex]Z = \frac{X - \mu}{\sigma}[/tex]

Z = (125 - 150)/25

Z = -1

Z = -1 has a p-value of 0.16, rounded with the Empirical Rule, which is the proportion.

The proportion with cholesterol levels between 200 and 225 mg/dl is the p-value of Z when X = 225 subtracted by the p-value of Z when X = 200, hence:

X = 225

[tex]Z = \frac{X - \mu}{\sigma}[/tex]

Z = (225 - 150)/25

Z = 3

Z = 3 has a p-value of 0.9985.

X = 200

[tex]Z = \frac{X - \mu}{\sigma}[/tex]

Z = (200 - 150)/25

Z = 2

Z = 2 has a p-value of 0.975.

0.9985 - 0.975 = 0.0235 = 2.35%, which is the percentage.

A similar problem, also involving the normal distribution, is given at https://brainly.com/question/4079902

#SPJ1

I have the area of the circle but having trouble find the area of the triangle

Answers

To calculate the area of the triangle we need the length of the base and the height, being the height perpendicular to the base.

The base of the triangle has a length that is equal to the diameter of the circle. It can also be expressed as 2 times the radius r. So the base is:

[tex]b=2\cdot r=2\cdot4=8\operatorname{cm}[/tex]

The height is the segment perpendicular to the base that goes up to the vertex at the top. as it goes from the center of the circle to the border of the circle, it has a length that is equal to the radius r:

[tex]h=r=4\operatorname{cm}[/tex]

Then, we can calculate the area of the triangle as:

[tex]A=\frac{b\cdot h}{2}=\frac{8\cdot4}{2}=\frac{32}{2}=16\operatorname{cm}^2[/tex]

We can calculate the area of the circle as:

[tex]A_c=\pi r^2\approx3.14\cdot4^2=3.14\cdot16=50.24[/tex]

The probability that a randomly selected point within the circle falls in the white area is equal to the ratio of white area to the area of the circle.

The white area is equal to the area of the circle minus the area of the triangle.

Then, we can calculate the probability as:

[tex]p=\frac{A_w}{A_c}=\frac{A_c-A_t}{A_c}=\frac{50.24-16}{50.24}=\frac{34.24}{50.24}\approx0.68=68\%[/tex]

Answer: The probability is p=0.68.

A length of 48 ft. gave Malama an area
of 96 sq. ft. What other length would
give her the same area (96 sq. ft.)?
4

Answers

I would say the answer is either 48 or 2. Whatever is on the multiple choice

My explanation:


Easy explanation ⬇️

Given length: 48ft

Total area is 96sq. ft

48 + 48 = 96


Second explanation:

Formula to find missing length ⬇️

Area = length x width

96 sq. ft = 48ft x w

96 sq. ft = 48ft x 2



(2 x 48 = 96)



So 2 (probably 48) should be your answer!

* Use the digits 0, 2, and 5 to write all of the three-digit numbers that fit each
description. You can repeat digits in a number.
multiples of 2

Answers

The 3-digit multiples of 2 using 0, 2, and 5 are:

250502520

What are multiples?A multiple in science is created by multiplying any number by an integer. In other words, if b = na for some integer n, known as the multiplier, it can be said that b is a multiple of a given two numbers, a and b. This is equivalent to stating that b/a is an integer if an is not zero. In mathematics, multiples are the results of multiplying an integer by a given number. Multiples of 5 include, for instance, 10, 15, 20, 25, 30, etc. Numerous 7s include 14, 21, 28, 35, 42, 49, etc.

So, 3-digit multiples of 2 using the digits 0, 2, and 5 are:

3 digits multiples of 2:

250502520

Therefore, the 3-digit multiples of 2 using 0, 2, and 5 are:

250502520

Know more about multiples here:

https://brainly.com/question/16054958

#SPJ13

Shawn needs to reach a windowsill that is 10 feet above the ground. He placed his ladder 4 feet from the base of the wall. It reached the base of the window.

a. Draw a diagram of the right triangle formed by Shawn's ladder, the ground and the wall.

b. Find the length of Shawn's ladder to the nearest tenth of a foot.​

Answers

If shown needs to reach a windowsill that is 10 feet above the ground and he placed his ladder 4 feet from the base of the wall.

Part a

The diagram of the right triangle formed by Shawn's ladder, the ground and wall has been plotted

Part b

The length of the Shawn's ladder is 10 foot

The distance between ladder base to the base of the wall = 4 feet

The distance between the wall base to the base of the window = 10 feet

Draw the right triangle using the given details

Part b

Using the Pythagorean theorem

[tex]AC^2= AB^2+BC^2[/tex]

Where AC is the length of the ladder

Substitute the values in the equation

AC = [tex]\sqrt{10^2+4^2}[/tex]

= [tex]\sqrt{100+16}[/tex]

= [tex]\sqrt{116}[/tex]

= 10.77

≈ 10 Foot

Hence, if shown needs to reach a windowsill that is 10 feet above the ground and he placed his ladder 4 feet from the base of the wall.

Part a

The diagram of the right triangle formed by Shawn's ladder, the ground and wall has been plotted

Part b

The length of the Shawn's ladder is 10 foot

Learn more about Pythagorean theorem here

brainly.com/question/14930619

#SPJ1

how to write the rule for the rotation on #11?

Answers

#11

If the point (x, y) is rotated 180 degrees around the origin clockwise or anti-clockwise, then its image will be (-x, -y)

We just change the sign of the coordinates

From the attached picture we can see

The parallelogram MNOP where

M = (1, -2)

N = (3, -2)

O = (4, -4)

P = (2, -4)

The parallelogram M'N'O'P' where

M' = (-1, 2)

N' = (-3, 2)

O' = (-4, 4)

P' = (-2, 4)

Since all the signs of the coordinates are changed, then

M'N'O'P' is the image of MNOP by rotation 180 degrees around the orign

The rule of transformation is

[tex]R\rightarrow(O,180^{\circ})[/tex]

A) What is the perimeter of the regular hexagon shown above?B) What is the area of the regular hexagon shown above?(see attached image)

Answers

Remember that

A regular hexagon can be divided into 6 equilateral triangles

the measure of each interior angle in a regular hexagon is 120 degrees

so

see the attached figure to better undesrtand the problem

each equilateral triangle has three equal sides

the length of each side is given and is 12 units

Part A) Perimeter

the perimeter is equal to

P=6(12)=72 units

Part B

Find the area

Find the height of each equilateral triangle

we have

tan(60)=h/6

Remember that

[tex]\tan (60^o)=\sqrt[]{3}[/tex]

therefore

[tex]h=6\sqrt[]{3}[/tex]

the area of the polygon is

[tex]A=6\cdot\lbrack\frac{1}{2}\cdot(6\sqrt[]{3})\cdot(12)\rbrack[/tex][tex]A=216\sqrt[]{3}[/tex]alternate way to find out the value of h

applying Pythagorean Theorem

12^2=6^2+h^2

h^2=12^2-6^2

h^2=108

h=6√3 units

State the domain of the function.{-2,0, 1, 2, 3, 4){-4,0, 1, 2, 6){0, 1,2,3)(-2,4)

Answers

D= {-2,0,1, 2,3,4}

1) Considering that the Domain is the set of entries of a function, on the x-axis, and examining that graph we can state

- The lowest value for that is given by x=-2

- The highest value for that is x= 4

- The points (-2,-4) (0,0), (1,1), (2,2), (3,1) and (4,6)

2) So, we can write the set, the Domain, after examining the options as:

D= {-2,0,1, 2,3,4}

Notice that we're considering the x-coordinates

3) So the answer is D= {-2,0,1, 2,3,4}

Yesterday, Diane had c baseball cards. Today, she gave 6 away. Using c, write and expression for the number of cards Diane has left.

Answers

Answer:

The expression is c-6. She gave away 6 cards so subtract 6 from the original number which is c.

9. SAILING The sail on Milton's schooner is the shape of a 30°-60°-90°triangle. The length of the hypotenuse is 45 feet. Find the lengths of thelegs. Round to the nearest tenth.

Answers

The triangle is shown below:

Notice how this is an isosceles triangle.

We can find the lengths of the hypotenuse by using the trigonometric functions:

[tex]\sin \theta=\frac{\text{opp}}{\text{hyp}}[/tex]

Then we have:

[tex]\begin{gathered} \sin 45=\frac{21}{hyp} \\ \text{hyp}=\frac{21}{\sin 45} \\ \text{hyp}=29.7 \end{gathered}[/tex]

Therefore the hypotenuse is 29.7 ft.

Three friends rented a kayak. It cost $4 per hour per person to rent the kayak, plus $2 for each life jacket, and $3 to park the car. It cost $57 in all. How many hours did they spend kayaking? Write an equation and solve.

Answers

Answer:

13 hours

Step-by-step explanation:

Let y = the total cost

let x = hours

y = 4x + 5      5 = the one time fee of the jacket  and the parking

57 = 4x + 5  Subtract 5 from both sides

52 = 4x  Divide both sides by 4

13 = x

The sides of an L-shaped figure meet all the right angles

Answers

ANSWER:

24 ft²

STEP-BY-STEP EXPLANATION:

To determine the area of the figure, we must divide the L-shaped figure into two rectangles just like this:

We calculate the area of each rectangle and the sum of both areas would be the area of the L-shaped figure, in the following way:

[tex]\begin{gathered} A_1=L\cdot W=6\cdot2=12\text{ ft}^2 \\ \\ A_2=L\cdot W=3\cdot4=12\text{ ft}^2 \\ \\ \text{ Therefore:} \\ \\ A_t=A_1+A_2=12+12 \\ \\ A_t=24\text{ ft}^2 \end{gathered}[/tex]

The area of the L-shaped figure is equal to 24 ft².

given which of the following describes the boundary line and shading for the second inequality in the system

Answers

Answer:

Solid Line, Shade Above

Explanation:

Given:

[tex]\left\{\begin{array}{l} y<-2 x+3 \\ y \geq x-4 \end{array}\right.[/tex]

The second inequality in the system is:

[tex]y\geq x-4[/tex]

The intercepts of the boundary line (y=x-4) are (0, -4) and (4,0).

Since the inequality has an equal to sign attached, we use a solid line.

At (0,0)

[tex]\begin{gathered} y\geq x-4 \\ 0\geq-4 \end{gathered}[/tex]

Since the inequality 0≥-4 is true, shade the side that contains (0, 0) as shown in the graph below:

So, we use a solid line and shade above the boundary line.

Graph the line by plotting any two ordered pairs with integer value coordinates that satisfy the equation.- 21 = 0AnswerKeypadKeyboard ShortcutsPoints can be moved by dragging or using the arrow keys. Any lines or curves will be drawn once allrequired points are plotted and will update whenever a point is moved.10SI10310101

Answers

We are given the following equation of a line.

[tex]-2x=0[/tex]

Let us first solve the above equation for x.

Divide both sides of the equation by -2

[tex]\begin{gathered} -2x=0 \\ \frac{-2x}{-2}=\frac{0}{-2} \\ x=0 \end{gathered}[/tex]

So, the solution is x = 0

This means that the two ordered pairs must contain the x-coordinate 0 and the y-coordinate can be any value you like.

For example:

(0, -5) and (0, 5)

Here the x-coordinate is 0 and the y-coordinate is -5 and 5.

Let us plot these ordered pairs and the line on the given graph.

Other Questions
Exam ContentQuestion 25Approximately how many years would it take money to grow from $5,000 to $10,000 if it could earn 6% interest? What is meant by differemtial outcomes in regards to Diversity-validity dilema Food Web AssignmentDirections illustrate the flow of energy within the aquatic system below by completing each direction below.SquidBlue WhaleSealKrillPenguinZooplankton1.) Draw in 8 arrows to show how the energy moves to create a food web. You may have to do a little research to help youmake all the correct connections!2.) Where would phytoplankton and the Sun belong in this food web? Draw it as well as an arrow connecting each one accurately.3.) How would bacteria nit into this food web? Describe how you would draw it in. 4.) Classify each of the 7 organisms in the aquatic food web, as well as bacteria, as a producer, consumer or decomposer by writing each into the correct category below. Producer - Consumer - Decomposer - Which of the following is usually performed by a scientist even though it is not considered a formal step in the scientific method? A. communicate results B. catalog informationC. discard data D. withhold findings Number of Senators every state hasGroup of answer choices42Depends on the size of the population Which of the following is only true sometimes? A. The sum of a rational number and a rational number is rational. B. The sum of a rational number and an irrational number is irrational. C. The product of an irrational number and an irrational number is irrational. D. The product of a nonzero rational number and an irrational number is irrational. -1514,2 30r2y3 + 45ryjent of517is 3(x^3)y + 6x(y^2) - 3.1. Whe3(x^3)y - 6x(y^2) +9Res-3(x^3)y + 6x(y^2) - 33(x^2)y + 5x(y^2) - 93(x^3)y + 5x(y^2) + 3 The four Creative Commons conditions include Attribution, ShareAlike,NonCommerical,and NoDerivatives.True or false? An apple is resting on a table. Claire says that there are no forces acting on the apple because it is not moving. Is she right? 4) The half-life of a medication is the amount of time for half of the drug to be eliminated from the body. The half-life of Advil or ibuprofen is represented by the equation 2 ) 5 . 0 ( t M R = , where R is the amount of Advil remaining in the body, M is the initial dosage, and t is time in hours. The density of copper is 8.94 g/cm^3. What is the mass of a rectangular sheet of copper: 10 cm wide, 45 cm long, and 0.2 cm thick? I have no idea how to solve for mass or where to start. :( Which expression is equivalent to (2-3x) (2+3x) ? what is the principal difference between a radio wave and light? what is the principal difference between a radio wave and light? radio waves have significantly higher frequencies than light. a radio wave is a mechanical vibration of matter and light is a vibration of electric and magnetic fields. radio waves have significantly lower frequencies than light. light is a mechanical vibration of matter and radio wave is a vibration of electric and magnetic fields. calculate the number of litres of each gas givin the following number of moles at STPhow many litres of gas is in 0.001 mol H2S 7-Which beans are the better deal? Kidney Beans $1.18 per lb O Lima Beans $213 for 2 lbs 76-What is the Unit Price for the better deal? Round to the nearest hundredth) Put your answer in the form 0.00 or .00, so if answer is 43 cents, its 0.43 or.43, if there is a dollar amount like 1.50, do not add zeros in front). down home furnishings reports inventory using the lower of cost and net realizable value (nrv). below is information related to its year-end inventory. inventory quantity unit cost unit nrv furniture 260 $91 $106 electronics 56 460 400 required: 1. calculate the total recorded cost of ending inventory before any adjustments. 2. calculate ending inventory using the lower of cost and net realizable value. 3. record any necessary adjusting entry for inventory. 4. determine the impact of the adjusting entry in the financial statements. What is the slope of this line? :( don is researching the seasonal flu vaccine for an informative speech and begins his search online with the centers for disease control. at what step is don in the speech preparation process? A restaurant offers a $12 dinner special that has 4 choices for an appetizer, 11 choices for an entree, and 3 choices for a dessert. How many different meals are available when you select an appetizer, an entree, and a dessert? Victoria, Cooper, and Diego are reading the same book for theirlanguage arts class. The table shows the fraction of the bookeach student has read. Which student has read the leastamount? Explain your reasoning.